7
$\begingroup$

A combinatorial game I am studying has given rise to the following question. Consider the group $\Bbb Z/n\Bbb Z$. What is the largest $m$ such that there exist $k$ sets of $m$ residues such that the intersection of a translation of each of these sets has at most 1 element? That is, if the sets are $A_1, \ldots A_k$, we require that for all $(c_1, \ldots, c_k)$ the intersection of $(A_i + c_i)$ for $1 \le i \le k$ has at most one element, where $A_i + c_i$ is set obtained by adding $c_i$ to every element in $A_i$. Alternatively, if it makes the answer simpler, we can ask what the smallest $n$ is given $m$ and $k$.

For $k=2$, the answer is simple. It is possible when $n\ge m^2$ by making one set $\{0,1,\ldots, m-1\}$ and the other $\{0,m,\ldots, m(m-1)\}$. But it's not clear to me how to extend the construction to $k\ge 3$.

If there is a simple generalization to the case where the sets of residues can be different sizes, I'd be interested in that as well. That is, we are given $k$ and $(m_1, \ldots, m_k)$, where the $i$-th set must have size $m_i$, and we are to find the smallest $n$ for which the sets can have a single mutual intersection.

Thanks in advance for any help.

$\endgroup$
4
  • 1
    $\begingroup$ It is necessary and sufficient that for any nonzero $d\in\Bbb Z/n\Bbb Z$, there exists $i\in\{1,2,\dots,k\}$ such that $d\notin (A_i-A_i)$. $\endgroup$ Aug 13, 2020 at 4:18
  • 1
    $\begingroup$ Have you tried k different coprime integers j_i?. Then for N less than the product of the j_i, the sets which are 0 mod j_i and have about N/j_i elements each should work. For N=200, the numbers 5,6, and 7 do a nice job. I'm not sure you will do much better than m=200/7 for k=3 and N=200. Gerhard "Residue Classes Can Be Shifty" Paseman, 2020.08.12. $\endgroup$ Aug 13, 2020 at 4:22
  • $\begingroup$ I think the sets work for $n\ge m$. I've edited my question to fix this. Thanks for pointing it out. $\endgroup$
    – BPP
    Aug 13, 2020 at 5:11
  • $\begingroup$ @GerhardPaseman: We should have $j_i$ as divisors of $n$ as otherwise "negative" differences would not be multiples of $j_i$. Ideally $n$ should be LCM of the $j_i$. For $k=3$ and $n=200$, we have with necessity that $m\leq 34$. See my answer for details. $\endgroup$ Aug 13, 2020 at 17:06

1 Answer 1

1
$\begingroup$

It is necessary and sufficient that for any nonzero $d\in\Bbb Z/n\Bbb Z$, there exists $i\in\{1,2,\dots,k\}$ such that $d\notin (A_i-A_i)$. In other words, $$\bigcap_{i=1}^k (A_i-A_i) = \{0\}.$$ This holds even for sets of varying sizes.


Since $| A_i-A_i|\geq |A_i| = m$, we get a necessary condition: $n-1\leq k(n-m)$, that is $$m\leq \frac{(k-1)n+1}k.$$ For varying set sizes, it is $$n\geq \frac{m_1+\cdots+m_k-1}{k-1}.$$


Another necessary condition can be obtained from the observation that for any $a\in\Bbb Z/n\Bbb Z$, there exist $m^k$ vectors $(c_1,\dots,c_k)$ such that $a\in \bigcap (A_i+c_i)$. Since these vectors must be distinct for distinct $a$, we have $n\cdot m^k\leq n^k$, that is $$m\leq n^{(k-1)/k}.$$ This condition implies that the given example for $k=2$ is optimal when $m^2\leq n<(m+1)^2$.

For varying set sizes, the last condition takes form: $$n\geq (m_1\cdots m_k)^{1/(k-1)}.$$


As for the construction, the following streamlining of Gerhard's idea does the job for a given $k$, although it's not necessarily optimal.

Take any integers $0<b_1<b_2<\dots<b_k$, set $n:=\mathrm{LCM}(b_1,\dots,b_k)$, $m:=\frac{n}{b_k}$, and select $A_i$ as any subset of $b_i(\Bbb Z/n\Bbb Z)$ with $|A_i|=m$. Indeed, in this setting, for any nonzero $d\in\Bbb Z/n\Bbb Z$, there exists $i\in\{1,2,\dots,k\}$ such that $b_i\nmid d$, implying that $d\notin (A_i-A_i)$. For Gerhard's example with $k=3$ and $b_1=5$, $b_2=6$, and $b_3=7$, we have $n=210$ and $m=30$.

If we are also given $n$, this construction becomes more tricky as we need to pick $k$ numbers $0<b_1<b_2<\dots<b_k$ with the smallest $b_k$ possible such that $n\mid \mathrm{LCM}(b_1,\dots,b_k)$. It is clear that $b_k$ cannot be smaller than the largest primepower dividing $n$. It also follows that here it does not make sense to have $k$ greater than the number of distinct primes dividing $n$, since $b_i$ being the distinct primepowers forming the prime factorization of $n$ do the job.

$\endgroup$

Your Answer

By clicking “Post Your Answer”, you agree to our terms of service and acknowledge that you have read and understand our privacy policy and code of conduct.

Not the answer you're looking for? Browse other questions tagged or ask your own question.